What is Inequality: Definition and 1000 Discussions

In mathematics, the triangle inequality states that for any triangle, the sum of the lengths of any two sides must be greater than or equal to the length of the remaining side. This statement permits the inclusion of degenerate triangles, but some authors, especially those writing about elementary geometry, will exclude this possibility, thus leaving out the possibility of equality. If x, y, and z are the lengths of the sides of the triangle, with no side being greater than z, then the triangle inequality states that




z

x
+
y
,


{\displaystyle z\leq x+y,}
with equality only in the degenerate case of a triangle with zero area.
In Euclidean geometry and some other geometries, the triangle inequality is a theorem about distances, and it is written using vectors and vector lengths (norms):






x

+

y





x


+


y


,


{\displaystyle \|\mathbf {x} +\mathbf {y} \|\leq \|\mathbf {x} \|+\|\mathbf {y} \|,}
where the length z of the third side has been replaced by the vector sum x + y. When x and y are real numbers, they can be viewed as vectors in R1, and the triangle inequality expresses a relationship between absolute values.
In Euclidean geometry, for right triangles the triangle inequality is a consequence of the Pythagorean theorem, and for general triangles, a consequence of the law of cosines, although it may be proven without these theorems. The inequality can be viewed intuitively in either R2 or R3. The figure at the right shows three examples beginning with clear inequality (top) and approaching equality (bottom). In the Euclidean case, equality occurs only if the triangle has a 180° angle and two 0° angles, making the three vertices collinear, as shown in the bottom example. Thus, in Euclidean geometry, the shortest distance between two points is a straight line.
In spherical geometry, the shortest distance between two points is an arc of a great circle, but the triangle inequality holds provided the restriction is made that the distance between two points on a sphere is the length of a minor spherical line segment (that is, one with central angle in [0, π]) with those endpoints.The triangle inequality is a defining property of norms and measures of distance. This property must be established as a theorem for any function proposed for such purposes for each particular space: for example, spaces such as the real numbers, Euclidean spaces, the Lp spaces (p ≥ 1), and inner product spaces.

View More On Wikipedia.org
  1. D

    Proving an Inequality with Elementary Methods

    Homework Statement Let 0 ≤ x_1 ≤ x_2 ≤ ... ≤ x_n and x_1 + x_2 + ... + x_n = 1 , All the 'x' are real and n is a natural number. Prove the following: (1+x_1^21^2)(1+x_2^22^2)...(1 + x_n^2n^2) ≥ \frac{2n^2+9n+1}{6n} Homework Equations The Attempt at a Solution...
  2. T

    Triangle Inequality in 'Linear Algebra Done Right'

    I'm stuck on one aspect of the proof on page 105 of the 2nd edition. Equation 6.13 is necessary for the inequality to be an equality as it says but they never seem to account for inequality 6.11. Specifically, I don't see how this satisfies 2 Re<u,v> = 2 |<u,v>| Thanks for any guidance.
  3. J

    Inequality integral absolute value derivative

    Here's a claim: Assume that a function f:[a,b]\to\mathbb{R} is differentiable at all points in its domain. Then the inequality |f(b) - f(a)| \leq \int\limits_{[a,b]}|f'(x)|dm(x) holds. The integral is the Lebesgue integral. Looks simple, but I don't know if this is true. There exists...
  4. morrobay

    What Interpretation/Model of QM Predicts Bell Inequality Violations ?

    Are there any modern interpretations of QM that predict the correlations in a Bell Inequality violation ? Preferably a local non realistic model based on mechanisms.
  5. C

    How can I prove the inequality relationship between RMS, AM, GM, and HM?

    Homework Statement I am asked to discover and prove the inequality relationship between root mean square, arithmetic mean, geometric mean, and harmonic mean. Homework Equations Let a,b, be non-negative integers. (a-b)2 ≥ 0 and (√a-√b)2 ≥ 0 The Attempt at a Solution Using (a-b)2 ≥...
  6. bohm2

    Legget's inequality and Bohmian mechanics

    Leaving aside the debatable point about whether bell's violation rules out all local theories or just local realism, is there general agreement that if the assumptions are valid, violation of Leggett's inequalities rules out any non-local model that treats properties other than position as real...
  7. N

    Solving Complex Inequalities with Absolute Value Signs

    [solved]Complex inequality Homework Statement You have the two inequalities, where k is a complex number; |k+\sqrt{k^2-1}|<1 and |k-\sqrt{k^2-1}| <1 Show that if ##|k|>1##, then the second inequality is fulfilled, while the first one is impossible for any value of k. The Attempt...
  8. S

    Proving an Inequality by Induction

    Homework Statement Question: Use induction to prove that 3^n > n x 2^n for every natural number n ≥ 3 Homework Equations N/A The Attempt at a Solution Answer: Step 1: 3^3 > 3 x 2^3 ⇒ 27 > 24 Step 2: Assume 3^k > k x 2^k Step 3: 3^(k+1) > (k+1) x 2^(k+1) ⇒ 3 x 3^k > k x 2^(k+1) +...
  9. binbagsss

    Thermodynamics, Entropy, Clausis Inequality, Reversible and Irreversib

    ∫dQ/T≤∫dQ(rev)/T * , where both integrals are evaluated between the same thermodynamic coordinates- A and B , say. - I am having trouble interpreting this inequality. -( I understand the derivation in my textbook via the Clausius diagram(considering a reversible and an ireversible process...
  10. Saitama

    Solve Inequality: f(x)=1-x-x3, 1-f(x)-f3(x)>f(1-5x)

    Homework Statement Let f(x)=1-x-x3. Find all the real values of x satisfying the inequality, 1-f(x)-f3(x)>f(1-5x).Homework Equations The Attempt at a Solution I honestly don't know how to start with this one. Substituting f(x) directly in the inequality doesn't look like a good idea. I need a...
  11. B

    Solving inequality with different power variables

    Homework Statement Solve for k: k2 - 16k < 0 In the answer it has 0 < k < 16, I do not know how they get there from the original question.
  12. K

    Proving summation series inequality

    Question http://puu.sh/52zAa.png Attempt http://puu.sh/52AVq.png I've attempted to use Riemann sums and use the integral to prove the inequality, not sure if this was the right approach to start with as I am now stuck and don't see what to do next. For part (b), I know that if (2√n...
  13. PsychonautQQ

    Proving Inequality: Double Integral (dA / (4+x^2+y^2)) ≤ π

    Homework Statement Prove the inequality double integral (dA / (4+x^2+y^2)) is less than or equal to pi, where the double integral has a sub D where D is the disk x^2 + y^2 less than or equal to four Homework Equations The Attempt at a Solution I really have no idea, anyone want to...
  14. J

    Proving |x + y| ≥ |x| - |y| using Theorem 3 and the fact that |-y| = |y|

    Homework Statement |x + y| ≥ |x| - |y| [Hint: write out x = x + y - y, and apply Theorem 3, together with the fact that |-y| = |y|] Homework Equations Theorem 3: |a + b| ≤ |a| + |b| x = x + y - y |-y| = |y| The Attempt at a Solution |x + y| ≥ |x| - |y| x = x + y - y (don't know where to...
  15. R

    Chebychev's inequality for two random variables

    (I wasn't sure how to title this, it's just that the statement resembles Chebychev's but with two RV's.) Homework Statement Let \sigma_1^1 = \sigma_2^2 = \sigma^2 be the common variance of X_1 and X_2 and let [roh] (can't find the encoding for roh) be the correlation coefficient of X_1 and X_2...
  16. N

    MHB Chebyshev Inequality: Get Expert Help Now

    Could I get some help with this question? Please refer to the attachment. Thanks
  17. C

    Derivation of Clausius Inequality

    I have been reading about the derivation of Clausius' Inequality and there are a few things I do not understand. I have attached an image of the cycles. B) shows one carnot engine performing work ##d W_i## per cycle and delivering heat ##d Q_i## per cycle. For ##T'## to remain unchanged, it...
  18. G

    Analyzing Bell's inequality for different measurement angles

    Hi, I'm trying get a better understanding of Bell's inequality in the form $$\left|E\left(\bf{a},\bf{b}\right) -E\left(\bf{a},\bf{c}\right)\right|\leq 1+E\left(\bf{b},\bf{c}\right)\enspace.$$ I'm considering the Bell state $$\left|\psi\right\rangle=...
  19. L

    The usefulness of proofs to a physicist: eg The Schwarz Inequality

    I'm trying to really solidify my maths knowledge so that I'm completely comfortable understanding why and how certain branches of mathematics are introduced in physics and inevitably that leads me to a study of proofs. I usually skip proofs as I found them annoying, unintuitive and just...
  20. skate_nerd

    MHB Inequality proof w/ induction, 2 unknowns

    I am given a statement to prove: Show (without using the Binomial Theorem) that \((1+x)^n\geq{1+nx}\) for every real number \(x>-1\) and natural numbers \(n\geq{2}\). I am given a hint to fix \(x\) and apply induction on \(n\). I started by supposing \(x\) is a fixed, real number larger than -1...
  21. B

    Variation of the triangle inequality on arbitrary normed spaces

    The following inequality can easily be proved on ##ℝ## : ## ||x|-|y|| \leq |x-y| ## I was wondering if it extends to arbitrary normed linear spaces, since I can't seem to prove it using the axioms for linear spaces. (I can however, prove it using the definition of the norm on ##ℝ## by using...
  22. W

    MHB Need Explanations For The Red Part (2 variable inequality)

    Prove that for any two numbers x,y we have [(x^2 + y^2)/2] >= x + y - 1 Solution) For any number a we have have a^2 > 0. So, (x-1)^2 + (y-1)^2 >= 0 And if we solve this we get the solution.I don't get the red part.
  23. S

    Proving Inequality using Arithmetic and Order Axioms

    Using only the axioms of arithmetic and order, show that: for all x,y satisfy 0≤x, 0≤y and x≤y, then x.x ≤ y.y I'm really lost on where to start, my attempt so far was this as 0 <= x and 0 <= y, we have 0 <= xy from axiom (for all x,y,z x<=y and 0<=z, then x.z <=y.z). then we use the...
  24. C

    Inequality absolute value help

    ## x - |x-|x|| > 2 ## how would I go about solving something like this? my initial thoughts was to consider if x >= 0 I get 2-x < 0 then x > 2 in that case then consider if x < 0 which I get -|x+x| > 2-x then 2x > 2-x then x > 2/3 but I'm having troubles deciding which one is correct, and if...
  25. S

    MHB Proof of inequality involving circular and hyperbolic trig. functions

    Hi guys, Can you help me I am stuck: By finding the real and imaginary parts of z prove that, $$|\sinh(y)|\le|\sin(z)|\le|\cosh(y)|$$ i have tried the following: Let $$z=x+iy$$, then $$\sin(z)=sin(x+iy)=\sin(x)\cosh(y)+i\sinh(y)\cos(x)$$ $$|\sin(z)|=\sqrt{(\sin(x)\cosh(y))^2+(\sinh(y)...
  26. C

    Discrete math sequence and inequality induction proof help

    Hello. I am reading an introduction to induction example, and I am having the hardest time trying to determine what exactly happened in the proof. Can somebody please help? How can ##3^{k-1}## + ##3^{k-2}## + ##3^{k-3}## all of a sudden become ##3^{k-1}##+##3^{k-1}##+##3^{k-1}## and how can be...
  27. F

    I found this inequality but I don't know where it comes from

    here is the inequality: ##(\sum\limits_{i=1}^n |x_i-y_i|)^2= \ge \sum\limits_{i=1}^n(x_i-y_i)^2+2\sum\limits_{i \neq j}^n |x_i-y_i|\cdot |x_j-y_j|## does it have a name/is the consequence of a theorem? Thank you :)
  28. A

    An inequality for a two variable function

    Suppose that ##F(u,v)=a|u+v|^{r+1}+2b|uv|^{\frac{r+1}{2}}##, where ##a>1, b>0## and ##r\geq3.## How we can show that there exists a positive constant c such that ## F(u,v)\geq c\Big( |u|^{r+1}+|v|^{r+1}\Big). ##
  29. NATURE.M

    Possible Values of the Inequality.

    Homework Statement What are the possible values of |2x−3| when 0<|x−1|<2? Homework Equations The Attempt at a Solution We know \left|x-1\right| becomes x-1 if x-1≥0 and -(x-1) if x-1<0. Now consider two cases. Case 1: 0<x-1<2 \Rightarrow 1<x<3 \Rightarrow -1<2x-3<3. Case...
  30. P

    Solving Logarithm Inequality Log x ((x+3)/(x-1)) > Log x x

    Log x ((x+3)/(x-1) > Log x x ?? I've managed to find 4 conditions for this inequality: 1. -1 > x > 3 2. x > -3 3. x > 0 4. x ≠ 1 but I'm not sure how to write the solution. Is it " 0 < x & 1 < 0 < 3 " ? Thanks.
  31. J

    Mathematical induction inequality

    Homework Statement Prove; n^2 > n+1 for n = 2,3,4 by Induction Homework Equations The Attempt at a Solution p(n)= P(2) 2^2> 2+1 --> 4>3 Induction step: P(n+1): (n+1)^2 > (n+1) +1 (n+1)^2> n+2 n^2 + 2n + 1 > n+2 | -n n^2 +n + 1> 2 | -1 n^2 +n > 1 Is this correct, and how...
  32. B

    How Does the Triangle Inequality Transform from Equality to Inequality?

    Hello all, I am currently reading about the triangle inequality, from this article http://people.sju.edu/~pklingsb/cs.triang.pdf I am curious, how does the equality transform into an inequality? Does it take on this change because one takes the absolute value of 2uv? Because before the...
  33. anemone

    MHB Inequality Challenge: Prove $x^x \ge (x+1/2)^{x+1}$ for $x>0$

    Prove x^x \ge \left( \frac{x+1}{2} \right)^{x+1} for $x>0$.
  34. morrobay

    Is Superposition the Explanation for Bell Inequality Violations ?

    This form of a Bell inequality: n[x-y+] + n[y-z-] ≥ n[x+z+] is derived from spin measurements at A and B when detector settings are aligned. If it is correct that when a particle is measured at detector A and is spin up in the y direction , then its entangled twin at B is in superposition...
  35. A

    Solve Inequality: x^2-4|-3|<1 | UofT Calculus

    Hey everyone. I'm taking Calculus at UofT and I got a question in a problem set that kind of got me thinking, and well, I'm not sure if I'm doing it correctly. This isn't the exact question, but, how would you go about solving this inequality: ||(x^2)-4|-3|< 1
  36. S

    MHB When does the floor function inequality hold?

    Let [x] be the floor function i.e. it produces the integral part of x. So for example if x = 1.5 then [x] = 1. I recently saw the claim [x] \geq x - 1 The strict part of the inequality makes perfect sense, but when does equality occur? Does it even occur at all? I have not been able to find an...
  37. S

    MHB Inequality Proof: Fun Problem | z,w <1 | Forum

    Here's a fun problem proof I came across. Show that \left| \frac { z- w }{1 - \overline{z}w} \right| < 1 given |z|<1, |w|<1. I attempted writing z and w in rectangular coordinates (a+bi) but to no avail. Any suggestions, forum?
  38. anemone

    MHB Can the Inequality Challenge be Proven: 2^{\frac{1}{3}}+2^{\frac{2}{3}}<3?

    Prove 2^{\frac{1}{3}}+2^{\frac{2}{3}}<3.
  39. M

    Trying to find delta in a limit involving an inequality

    Homework Statement Find δ (which is the input tolerance): lim (9 - 4x2)/(3 + 2x2) = 6 x→-1.5 Homework Equations |f(x) - L| < \epsilon |x - a| < \delta The Attempt at a Solution lim (9 - 4x2)/(3 + 2x2) = 6 x→-1.5 I need to get to: |x-(-1.5)| < \delta = |x + 1.5)| <...
  40. P

    Proving Inequality for Math Students

    Homework Statement Prove If ## 0 \leq a < b ## and ## 0 \leq c < d ## then ## ac < bd ## The Attempt at a Solution not sure how to even start on this, was thinking if a = 0 or c = 0, then ac = 0, but bd > 0 (which is given) so bd > ac however this seems like I'm cheating because they give...
  41. Albert1

    MHB Finding a Solution to an Inequality in Natural Numbers

    $a,b,c,d,e,f,g \in N$ $a<b<c<d<e<f<g$ $\dfrac{1}{a}+\dfrac{1}{b}+\dfrac{1}{c}+\dfrac{1}{d}+\dfrac{1}{e}+\dfrac{1}{f}+\dfrac{1}{g}=1$ please find one possible solution of a,b,c,d,e,f,g (you should find it using mathematical analysis,and show your logic,don't use any program)
  42. P

    How is the inequality |a+b| \leq |a| + |b| proven using different methods?

    prove |a+b| \leq |a| + |b| i've proved it considering all the 4 cases for a and b but the book went about it a different way: (|a+b|)^2 = (a+b)^2 = a^2 + 2ab + b^2 \leq a^2 + 2|a||b| + b^2 = |a|^2 + 2|a||b| + |b|^2 = (|a|+|b|)^2 it then goes on the conclude that |a+b| \leq |a| +...
  43. Seydlitz

    The inequality which implies f(x) > 0 - Spivak's Calculus

    Hello, if you guys would turn to page 117 in Spivak's Calculus, there is the proof for theorem 3. At the last line he stated that this last inequality ##|f(x)-f(a)|<f(a)## implies ##f(x)>0##. How can you check this fact? Can we assume first that ##f(x)-f(a)<0## to eliminate the absolute...
  44. Q

    How can I solve the inequality (-3/x) < 3?

    Homework Statement (-3/x) < 3 Homework Equations Dividing/multiplying an inequality causes the inequality sign to change. The Attempt at a Solution I keep getting the wrong solution. I tried two methods. I cannot get the textbook solution (x < -1) Method one: -3 < 3x...
  45. naima

    Bell's inequality when efficiency < 1

    Hi PF Somebody gave me this link could you help me to understand why (9) has to be inserted in (7) when the efficiency \eta < 1?
  46. N

    Trignometric inequality problem

    Homework Statement Given that -pi < x < pi, solve the following inequality in radians root(2) - 2sin(x-(pi/3)) < 0 The Attempt at a Solution root(2) - 2sin(x-(pi/3)) < 0 - 2sin(x-(pi/3)) < -root(2) sin(x-(pi/3)) > (root(2)) -pi<x<-pi -pi - (pi/3) < x - pi/3 < pi...
  47. U

    Proving Inequality: Vector Method for Cos2A+Cos2B+Cos2C in Triangle ABC

    Homework Statement In a triangle ABC, prove by vector method cos2A+cos2B+cos2C≥ -3/2. Homework Equations The Attempt at a Solution I can change the LHS of the inequality to the form (cos2A i + cos2B j + cos2C k).(i+j+k)
  48. E

    Is There a Non-Calculus Method to Prove this Thermodynamic Inequality?

    Prove that $$ \frac{y^x-1}{xy^{x-1}(y-1)}<1$$ where x,y \in ℝ, x>1 and y>1. I was able to prove it using calculus, but am wondering if there was another way of doing so, like exploiting some inequality-theorems which involves real numbers. I'll be glad if anyone can show me a way and...
  49. M

    Is a<-24 the Correct Solution to the Inequality Problem?

    Homework Statement ##\frac{a}{4}>\frac{a}{2}+6## The Attempt at a Solution ##\frac{2a}{2}>\frac{4a+48}{2}## ##a>2a+24## So do I just plug random numbers in and see what I get? I realized right away that it has to be a negative number so I stuck in -30 and got ##-30>-60+24## Well that's...
Back
Top